LSAT and Law School Admissions Forum

Get expert LSAT preparation and law school admissions advice from PowerScore Test Preparation.

 Administrator
PowerScore Staff
  • PowerScore Staff
  • Posts: 8916
  • Joined: Feb 02, 2011
|
#41660
Complete Question Explanation
(The complete setup for this game can be found here: lsat/viewtopic.php?t=8572)

The correct answer choice is D)

The local condition appears impossible to satisfy in any template (undoubtedly causing panic among some test-takers), until you realize that the second program is not necessarily the program given at 1:30! If the hour-long program, G, is given first, then the second program would begin at 2:00. The only two templates that would make this possible are Templates 1A and 1B (Template 1A would need a slight modification, as shown below):
PT65_D11 LG Explanations_game_#4_#19_diagram 1.png
With these solutions in place, you can easily determine what cannot be true by the process of elimination.

Answer choice (A): This answer choice is incorrect, because S could be the third program (Template 1B). In that template, the third program begins at 2:30, after G and R.

Answer choice (B): This answer choice is incorrect, because S could be the fourth program (Template 1A). In that template, the fourth program begins at 3:00, after G, R, and T.

Answer choice (C): This answer choice is incorrect, because T could be the fifth and last program in Template 1B.

Answer choice (D): This is the correct answer choice, because W is either the fourth or the fifth program in Template 1A, and it is the fourth program in Template 1B. Therefore, W cannot be the third program.

Answer choice (E): This answer choice is incorrect, because W could be the fifth and last program in Template 1A.
You do not have the required permissions to view the files attached to this post.
 crharke42
  • Posts: 12
  • Joined: Aug 26, 2012
|
#5007
#19 is still giving me troubles...


Putting R as the second program these are the two situations I came up with

T R G1 G2 S/W W/S

T R W/S S/W G1 G2

It says the answer is D... "Waterloo is the third program" but in the second situation I show that working. I think Answer C is correct.
User avatar
 Dave Killoran
PowerScore Staff
  • PowerScore Staff
  • Posts: 5853
  • Joined: Mar 25, 2011
|
#5009
You're really close, but both your scenarios have a fatal flaw: T is first (meaning it starts on the hour), and that violates the second rule.

The two scenarios that work for this question are:

..... ..... ..... G1 G2 R S W T

..... ..... ..... G1 G2 R T S/W W/S

So, (C) could occur, but (D) cannot occur, and hence (D) is correct.

Answering this type of question is where the templates really come in handy, because you can quickly narrow it down to these two scenarios.

Check those out and please let me know if they solve the problem. Thanks!
 crharke42
  • Posts: 12
  • Joined: Aug 26, 2012
|
#5015
Thank you! I don't know how I missed that!


I really appreciate it
 cfu1
  • Posts: 15
  • Joined: Mar 28, 2017
|
#34910
Dave, how can we assume that G will be shown at 1PM? Couldn't G still be shown at 2PM or 3PM? I'm confused as to how to approach this problem.

Thanks for your help!
 AthenaDalton
PowerScore Staff
  • PowerScore Staff
  • Posts: 296
  • Joined: May 02, 2017
|
#35073
Hi cfu,

Here's how we can deduce that program G is shown first in question 19.

If R is the second program and G (which takes up two spots) is not in spaces 1 and 2, we would have a linear board that looks like this:

_ R _ _ _ _

Under this hypothetical, we need to find space for the [G1G2] block in either spaces 3/4 (the 2 pm slot) or 5/6 (3 pm). Program T cannot go in the first space (1 pm) because of the rule which states that program T always starts at the half-hour. That means that T must immediately after W, either in the 2:30 or 3:30 spot. This creates a [WT] block.

At this point, the board would look either like this:

_ R G1 G2 W T

or

_ R W T G1 G2

Both of these scenarios would force S into space 1, before R. This would violate the R -- S rule.

Therefore, the only space the [G1G2] block can go in this question without forcing a rule violation is in the 1 pm slot.

Hope this makes sense!

Athena Dalton
 cfu1
  • Posts: 15
  • Joined: Mar 28, 2017
|
#35545
Sorry for the late response, but thanks for the explanation! Makes sense now. :)

Get the most out of your LSAT Prep Plus subscription.

Analyze and track your performance with our Testing and Analytics Package.